Вы находитесь на странице: 1из 9

MH1100/MTH112: Calculus I.

Solutions to the Problem list for Week #4.

Problem 1: (Based on #1.7.30 from [Stewart])


In this problem well prove the limit limx!2 (x2 + 2x
(i) Find a number C such that if 0 < |x

7) = 1.

2| < 1, then |x + 4| < C.

(ii) Use your answer to part (i) to write down a formal proof of this limit.
Solution
We are asked for a number C such that |x + 4| < C whenever 0 <
|x 2| < 1. Well think about this in two ways. First, consider a graph
of the function f (x) = |x + 4| restricted to the interval (1, 3)/ {2} (in other
words, restricted to the set {x 2 R|0 < |x 2| < 1}).

We can read from this graph that C = 7 will be a choice. (In fact, any
number C 7 is a correct choice.)
A more algebraic way to get the same answer is to use the triangle
inequality to deduce:
|x + 4| = |x
|x

2 + 6|
2| + 6

< 1+6
= 7.

(Because we are restricting to 0 < |x

2| < 1.)

Proof that limx!2 (x2 + 2x

7) = 1.

Let be an arbitrary positive real.


Set = min 1, 7 . (?)
Let x be an arbitrary x such that 0 < |x
Then:
|(x2 + 2x

7)

1| =
=
=
<

|x2 + 2x 8|
|(x + 4)(x 2)|
|x + 4||x 2|
|x + 4|

2| < .

(??)

(By equation (??).)

< 7

( 1, so 0 < |x 2| < 1,
so we can use our answer
to part (i).)

(By equation (?).)

Problem 2: (Based on #1.7.33 from [Stewart])


(i) Show that if 0 < |x

3| < min 1, 7 , then |x2

9| < .

(ii) Show that if 0 < |x

3| < min 2, 8 , then |x2

9| < .

(iii) Show that if 0 < |x

3| < min 2012, 2018


, then |x2

9| < .

We could do all three parts at once, but in the interest of simplicity we will
separate them. At the start of each part we will estimate the term |x + 3|.
This estimation will be used in the deduction that follows.
Proof of (i).
Let x satisfy 0 < |x

3| < min 1, 7 .

This implies that |x 3| < 1, so we can deduce (using the triangle inequality)
that:
|x + 3| = |x 3 + 6| |x 3| + 6 < 1 + 6 = 7 (?).
Thus:
|x2

9| =
<
<
=

|x 3||x + 3|
|x 3| 7
7 7
.

(By equation (?).)


(Because 0 < |x 3| < min 1, 7 .)

Proof of (ii).
Let x satisfy 0 < |x

3| < min 2, 8 .

This implies that |x 3| < 2, so we can deduce (using the triangle inequality)
that:
|x + 3| = |x 3 + 6| |x 3| + 6 < 2 + 6 = 8 (?).
Thus:
|x2

9| =
<
<
=

|x 3||x + 3|
|x 3| 8
8 8
.

(By equation (?).)


(Because 0 < |x 3| < min 2, 8 .)

Proof of (iii).
Let x satisfy 0 < |x

This implies that |x


inequality) that:
|x + 3| = |x

3| < min 2012, 2018


.

3| < 2012, so we can deduce (using the triangle


3 + 6| |x

3| + 6 < 2012 + 6 = 2018

(?).

Thus:
|x2

9| =
<
<
=

|x 3||x + 3|
|x 3| 2018

2018 2018
.

(By equation (?).)

(Because 0 < |x 3| < min 2012, 2018


.)

Problem 3: (#1.7.32 from [Stewart])


Prove that limx!2 x3 = 8.
Proof
Let be an arbitrary positive real.

Set = min( 19
, 1).
Let x be an arbitrary real such that 0 < |x
Then:
|f (x)

8| =
=

<

|x3 8|
|x 2||x2 + 2x + 22 |
|x 2|(|x|2 + 2|x| + 4)
(32 + 2 3 + 4)|x 2|

< (19)
.

2| < .

(By the triangle inequality.)


(Because when 0 < |x 2| < 1,
then |x| < 3.)

Problem 4: (#1.7.37 from [Stewart])


Let a > 0. Prove that limx!a

x=

a.

Proof
Let be a positive real number.
p
Set = a .
Let x be an arbitrary number from the domain of
Then:
p
| x

a| =

p
( x

a)

p
p
( x)2 ( a)2
p p
x+ a
p 1 p |x
a|
x+ a
p1 |x
a|
a

<

p1
a

=
=

x with 0 < |x

a| < .

p p
px+pa
x+ a

(This is an estimation step


1p
using px+
p1a for x 0.)
a

= .

Problem 5? :
1

Let a > 0. Prove that limx!a x 4 = a 4 .


Solution
This proof is obviously going to be some generalization of the previous
problem. The task is to understand how to generalize it. What we need is
1
1
some expression in x 4 and y 4 such that:
1

1
x 4 a 4 (Some expression.) = (x a).
The trick is to exploit the following standard identity, which is true for any
reals a, b 2 R and any integer p > 0:
b)(ap + ap

(a

b + ap

2 2

b + . . . + abp

+ bp ) = ap+1

bp+1 .

Note that this identity can be proved by induction.


Using this identity we can now write down a proof:
Proof:
Let be an arbitrary positive real number.
3

Set

= a 4 .
1

Let x be an arbitrary real from the domain of x 4 such that 0 < |x

a| < .

Then:
x

1
4

1
4

=
=

<

1
4

1 3

x4

a4

|x

1
4

1 3

1 2 1 1 1 2 1 3
+ x4
a4 + x4 a4 + a4
1 3 1 2 1 1 1 2 1 3
x4 + x4
a4 + x4 a4 + a4
x4

1 2 1 1 1 2 1 3
+ x4
a4 + x4 a4 + a4

x4

a4

a|

a4

= .

Problem 6: (#1.7.42 from [Stewart])


Prove that limx!

1
3 (x+3)4

= 1.

Proof
Let M be an arbitrary positive real number.
Set = 11 .
M4

Let x be an arbitrary number such that 0 < |x


Then:
f (x) =
=
>
=

( 3)| < .

1
(x + 3)4
1
|x ( 3)|4
1
4

1
1

M4

= M.

Problem 7? : (#1.7.43 from [Stewart])


Prove that limx!

5
(x+1)3

1.

Solution
According to the definition of a limit from the left, the x we will need to
test during the proof will be the set of x satisfying 1
< x < 1. The
challenging thing will be to use this information to find some inequality
5
relating the expression f (x) = (x+1)
. One
3 to some expression involving
approach is to make the following deductions:
1
)
)
)
)
)
)

<x<

1,

< 1 + x < 0,
>

(1 + x) > 0,
1
1
> > 0,
(1 + x)

3 3
1
1
>
> 0,
1+x
5
5
> 3 > 0,
3
(x + 1)
5
5
<
< 0. (?).
3
3
(x + 1)

Proof:
Let N be an arbitrary negative real number.

1
3
Set = 5N
(??).
Let x be an arbitrary real number such that 1
Then:
f (x) =

<x<

1.

5
,
(x + 1)3

<
= N

5
3

(Using equation (?).)


(Using equation (??).)

Problem 8: (Proof of the triangle inequality)


Solution to (i).
Take arbitrary numbers x, y 2 R. By fact #1 we deduce:

|x| x |x|

|y| y |y|

If we add these two inequalities we deduce:


(|x| + |y|) x + y (|x| + |y|).
Fact #2 now implies that |x + y| |x| + |y|.
Solution to (ii).
Using the triangle inequality we deduce:
|x| = |x

y + y|

|x

y| + |y|.

Rearranging this we deduce:


|x

|x|

y|

|y|.

From this statement we can also immediately deduce that


|x

y| = |y

x|

|y|

|x|.

Now | |x| |y| | either equals |x| |y| or |y| |x| (depending on whether
|x|
|y| or |y|
|x|), so together the previous two equations imply the
required fact:
|x y| | |x| |y| | .

Вам также может понравиться